Difference between revisions of "2005 AMC 12B Problems/Problem 23"

($LaTeX$-ified)
m (Solution)
Line 15: Line 15:
  
 
== Solution ==
 
== Solution ==
Call <math>x + y = s</math> and <math>x^2 + y^2 = t</math>.  Then, we note that log(s)=z which implies that <math>\log(10s) = z+1= \log(t)</math>.  Therefore, <math>t=10s</math>.  Let us note that <math>x^3 + y^3 = \frac[3st}{2}-\frac{s^2}{2} = s(15s-\frac{s^2}{2})</math>.  Since <math>s = 10^z</math>, we find that <math>x^3 + y^3 = 15\times10^2 - (1/2)\times10^3</math>. Thus, <math>a+b = \frac{29}{2}</math>.  <math>\boxed{\text{B}}</math> is the answer.
+
Call <math>x + y = s</math> and <math>x^2 + y^2 = t</math>.  Then, we note that <math>\log(s)=z</math> which implies that <math>\log(10s) = z+1= \log(t)</math>.  Therefore, <math>t=10s</math>.  Let us note that <math>x^3 + y^3 = \frac{3st}{2}-\frac{s^2}{2} = s(15s-\frac{s^2}{2})</math>.  Since <math>s = 10^z</math>, we find that <math>x^3 + y^3 = 15\times10^2 - (1/2)\times10^3</math>. Thus, <math>a+b = \frac{29}{2}</math>.  <math>\boxed{\text{B}}</math> is the answer.
  
 
== See Also ==
 
== See Also ==
  
 
{{AMC12 box|year=2005|ab=B|num-b=22|num-a=24}}
 
{{AMC12 box|year=2005|ab=B|num-b=22|num-a=24}}

Revision as of 13:08, 3 June 2011

Problem

Let $S$ be the set of ordered triples $(x,y,z)$ of real numbers for which

\[\log_{10}(x+y) = z \text{ and } \log_{10}(x^{2}+y^{2}) = z+1.\] There are real numbers $a$ and $b$ such that for all ordered triples $(x,y.z)$ in $S$ we have $x^{3}+y^{3}=a \cdot 10^{3z} + b \cdot 10^{2z}.$ What is the value of $a+b?$

$\textbf{(A)}\ \frac {15}{2} \qquad  \textbf{(B)}\ \frac {29}{2} \qquad  \textbf{(C)}\ 15 \qquad  \textbf{(D)}\ \frac {39}{2} \qquad  \textbf{(E)}\ 24$

Solution

Call $x + y = s$ and $x^2 + y^2 = t$. Then, we note that $\log(s)=z$ which implies that $\log(10s) = z+1= \log(t)$. Therefore, $t=10s$. Let us note that $x^3 + y^3 = \frac{3st}{2}-\frac{s^2}{2} = s(15s-\frac{s^2}{2})$. Since $s = 10^z$, we find that $x^3 + y^3 = 15\times10^2 - (1/2)\times10^3$. Thus, $a+b = \frac{29}{2}$. $\boxed{\text{B}}$ is the answer.

See Also

2005 AMC 12B (ProblemsAnswer KeyResources)
Preceded by
Problem 22
Followed by
Problem 24
1 2 3 4 5 6 7 8 9 10 11 12 13 14 15 16 17 18 19 20 21 22 23 24 25
All AMC 12 Problems and Solutions